Proving that this integral is divergent

  • #1
cbarker1
Gold Member
MHB
348
23
TL;DR Summary
I have a particular integral that I need to show that it cannot be finite in L1(positive real line) space. The integral comes from an Euler-Cauchy ODE. I want to show that the answer is not bounded.
Dear everyone,

I have a question on how to show that an integral is divigent. Here is the setup:
Suppose that we have the following function ##\sigma(x)=\frac{1}{x^{2-\varepsilon}}## for an arbitrary fixed ##\varepsilon>0.##
\begin{equation}
\dfrac{d}{dx}[-u'(x)]=\dfrac{1}{x^{2-\varepsilon}}u
\end{equation}
We want to verify using the bounds and using explicit form of the solution ##u##.

First we can rewrite the equation:
\begin{equation}
-u''(x)=u(x)\dfrac{1}{x^{2-\varepsilon}}
\end{equation}
Then we define a solution form of the differential equation ##u=x^{r-\varepsilon}## such that ##u## is in ##C^{2}(0,\infty)##, where ##C^{2}(0,\infty)## is the set of all continuously differentiable functions on ##(0,\infty).## Then we know that ##u''=(r-\varepsilon)(r-\varepsilon-1)x^{r-\varepsilon-2}##. Thus we have the following differential equation:
\begin{equation}
(r-\varepsilon)(r-\varepsilon-1)x^{r-\varepsilon-2}\cdotp x^{2-\varepsilon}+x^{r-\varepsilon}=0.
\end{equation}
Since #\varepsilon# is arbitrary, we can simplify the equation and factor ##x^{r-\varepsilon}## for ##x^{r-\varepsilon}>0;## therefore, we have the simplified equation:
\begin{equation}
(r-\varepsilon)(r-\varepsilon-1)+1=0.
\end{equation}
Now we must solve for ##r## in the auxiliary equation. We will expand the left-hand side and use the quadratic formula for ##r##:
\begin{equation}
r=\dfrac{2\varepsilon+1\pm \sqrt{(2\varepsilon+1)^2-4\bigg(\dfrac{(2\varepsilon+1)^2+3}{4}\bigg)}}{2}
\end{equation}
After some simplifications on the previous equations, we have the following solution:
## r=\dfrac{2\varepsilon+1\pm i\sqrt{3}}{2}.##
Now the solution to the differential equation is
##u(x)=x^{r-\varepsilon}## which implies that ##u(x)=c_1\sqrt{x}\cos(\frac{\sqrt{3}}{2}\log x)+c_2\sqrt{x}\sin(\frac{\sqrt{3}}{2}\log x)##. So we apply the one of the boundary condition that ##u(0)=0##; thus, the final solution is ##u(x)=\sqrt{x}\cos(\frac{\sqrt{3}}{2}\log x).## Now we need to check if the solution is not bounded.
In other words,
\begin{equation}
\sqrt{x}\cos\bigg(\frac{\sqrt{3}}{2}\log x\bigg)\geq \bigg(\int_{0}^{x}t^{-\varepsilon}dt\bigg)\bigg(\int_{x}^{\infty}t^{-\frac{3}{2}+\varepsilon}\cos(\frac{\sqrt{3}}{2}\log(t))dt\bigg)\\
\end{equation}
The red second integral is where I am having the most trouble on showing that it is not bounded.
Any suggestions are helpful. I thought to use a contradiction and squeeze theorem to show that the integral is not bounded.

Thanks,
Cbarker1
 
Last edited:
Physics news on Phys.org
  • #2
t:= log x, the given quantity, say f(t), is
[tex]e^{\frac{t}{2}} cos( \frac{\sqrt{3}}{2}t)[/tex]
It would make an estimation easier.
For any given number ##A>\frac{4\pi}{\sqrt{3}} =7.25...## there exists number a such that ##A<a<2A,\ |cos( \frac{\sqrt{3}}{2}a)|=1. ## so
[tex]|f(a)|=e^{\frac{a}{2}} > e^{\frac{A}{2}} > A [/tex]
unbounded.
 
Last edited:
  • #3
cbarker1 said:
... the final solution is ##u(x)=\sqrt{x}\cos(\frac{\sqrt{3}}{2}\log x).## Now we need to check if the solution is not bounded.

Take a sequence [itex]x_n[/itex] where [itex]\cos(x_n) = 1[/itex]; solving [itex]\frac{\sqrt{3}}{2}\log x = 2n\pi[/itex] gives [tex]x_n = \exp\left( \frac{4n\pi}{\sqrt{3}} \right).[/tex] Now [tex]
u(x_n) = \sqrt{x_n}\cos(2n\pi) = \exp\left( \frac{2n \pi}{\sqrt{3}} \right)[/tex] increases without bound as [itex]n \to \infty[/itex].

In general, if you want to find how [itex]g(x) \cos f(x)[/itex] or [itex]g(x) \sin f(x)[/itex] behave as [itex]x \to \infty[/itex], look at those values of [itex]x[/itex] for which [itex]f(x) = n\pi[/itex] (for cos) or [itex](n + \frac12)\pi[/itex] (for sin). These provide bounds for the extrema of the oscillation.
 
Last edited:

Similar threads

Replies
2
Views
1K
  • Set Theory, Logic, Probability, Statistics
Replies
1
Views
1K
  • Topology and Analysis
Replies
2
Views
1K
  • Calculus and Beyond Homework Help
Replies
9
Views
2K
  • Thermodynamics
Replies
7
Views
1K
  • Differential Equations
Replies
1
Views
882
Replies
8
Views
2K
  • Special and General Relativity
Replies
9
Views
1K
Replies
4
Views
943
  • Topology and Analysis
Replies
1
Views
1K
Back
Top